Cantidad de Movimiento e Impulso - Ejercicios Resueltos - Nivel 3

Поділитися
Вставка
  • Опубліковано 5 вер 2024
  • Veamos los problemas propuestos y ejercicios resueltos de cantidad de movimiento e impulso.
    ✔ Todos los videos de cantidad de movimiento e impulso: • Cantidad de Movimiento...
    Hoy vamos a revisar los ejercicios resueltos de cantidad de movimiento e impulso. El tema de choques y colisiones lo dejamos para el siguiente capítulo, porque es demasiado.
    En el nivel 1: veremos un poco de teoría y la explicación de ambos conceptos, cantidad de movimiento e impulso, también veremos sus fórmulas, y como trabajar de manera vectorial.
    En el nivel 2: veremos el principio de conservación de la cantidad de movimiento, así como algunos ejercicios interesantes, combinados con el tema de conservación de la energía mecánica.
    En el nivel 3: terminamos con un problema complicado, que demandará toda nuestra atención, se trata de un ejercicio de explosiones.
    Regreso la otra semana con un nuevo tema, ¡no olvides suscribirte!
    _______________________________________________________________
    ✔ Suscríbete: goo.gl/3HP9QH (no olvides darle like 😉)
    ✔ Guía de ejercicios y solución a la tarea: matemovil.com/...
    ✔ FACEBOOK: / matemovil
    ✔ INSTAGRAM: / matemovil2
    ✔ TWITTER: / matemovil1
    ✔ Conviértete en patrocinador: goo.gl/9mrmg9
    #impulsomatemovil

КОМЕНТАРІ • 108

  • @deborasuazo8717
    @deborasuazo8717 3 роки тому +11

    Muchísimas gracias por tus videos! Aprendí más de ti en unas horas que de mi maestro en 2 años

  • @snakersachock36
    @snakersachock36 7 років тому +64

    Viejo tus vídeos son geniales y se entienden bien. sigue haciendolo

    • @Matemovil
      @Matemovil  7 років тому +7

      Gracias Snakers, un saludo!

  • @nilonbender4472
    @nilonbender4472 3 роки тому +3

    Me cae muy bien señor, explica de maravilla, ya quisiera yo tener a un profesor como usted

  • @carlosoribe6637
    @carlosoribe6637 3 роки тому +8

    no este canal es buenisimo empezamos con pokebolas y de un momento a otro y sin darme cuenta estabamos con constantes universales y parace que cada vez se pone mas entretenido muchsimas gracias por tu trabajo

  • @ZioN.93
    @ZioN.93 6 років тому +2

    Viejo eres único explicando por favor sigue así adelante,Me has ayudado muchísimo.

  • @richardrl5914
    @richardrl5914 6 років тому +2

    Los problemas que pone en sus videos son realmente geniales, gracias por su orientación.

  • @nicolasmtz4341
    @nicolasmtz4341 7 років тому +7

    Te felicito por tu excelente trabajo en explicar los temas, te agradezco mucho por tener la visión de subir videos para explicar temas de fisica, mate, etc. Me sirvió mucho en estos dos días que los ví y ahora me siento preparado para mi examen de mañana. Te agradezco mucho, y te mando saludos desde méxico Nuevo León. Dios tb Jorge!

  • @user-tn9hf8ec6w
    @user-tn9hf8ec6w 3 роки тому +2

    Dentro de 2 semanas tengo un examen de este tema y con esta explicación estoy entendiendo más GRACIAS!!

  • @denisseguerrero8774
    @denisseguerrero8774 4 роки тому +1

    Muchísimas gracias muy buena explicación gracias a usted al fin entendí el tema

  • @alejandrodavidbenolol
    @alejandrodavidbenolol 6 років тому +10

    Estoy entendiendo, las pokebola eran para empezar y la granada era para matarnos. we fue genial lo de la constante universal. yo ni me imaginaba para que planteaba las componentes.

  • @carolinaashqui844
    @carolinaashqui844 Рік тому +2

    Hola, muchas gracias por tus explicaciones, son de mucha ayuda, solo tengo una duda, cómo sé cuándo la fuerza resultante es igual a cero?

  • @miltonwilliantorrejonmejia1926
    @miltonwilliantorrejonmejia1926 6 років тому +1

    Literal, eres genial,te felicito.

  • @lonelystar1214
    @lonelystar1214 3 роки тому +2

    Cómo se si la cantidad del movimiento se conserva o no?. O siempre se conserva?🤔.

  • @percysuasnabarmejia9346
    @percysuasnabarmejia9346 7 років тому

    Amigaso Jorge tus vídeos son muy buenos. muchos exitos

  • @jorgealverto4539
    @jorgealverto4539 2 роки тому +1

    Mi comentario es de agradecimiento por tu facilidad de trasmitir conocimiento abstracto
    Pero necesito saber que tema estudiar para saber cuanto peso puede hacer girar en un eje central una fuerza de un caballo de fuerza y la diferencia entre un eje vertical u horizontal y también en al variar la fuerza aplicada para encontrar la mejor forma de aprobechar la fuerza de trabajo resultante
    Se que tal vez no deje claro lo que pido pero si te es posible darme una pista te lo agradeceré mucho
    Te felicito por tu trabajo hasta luego

  • @DehelmisjoseRojasAngosto
    @DehelmisjoseRojasAngosto 7 років тому +1

    gracias profesor entendí 100%

  • @johanvera5795
    @johanvera5795 7 років тому +2

    matemovil realiza ejercicios de dinamica de rozamiento y tambien de hidrostatica. Please

  • @diegocalle571
    @diegocalle571 4 роки тому

    amigo sos un crack todo esta muy claro y el audio del video buenisimo tambien

  • @kevinbustos615
    @kevinbustos615 7 років тому +1

    grasia amigo tus videos me ayudan mucho

    • @Matemovil
      @Matemovil  7 років тому

      Que bien Kevin!! Un saludo a la distancia!

  • @kamilajinett1582
    @kamilajinett1582 6 років тому +2

    me mataste con este ejercicio , pero gracias por tus videos me ayudan mucho

  • @diamelavizcarra7317
    @diamelavizcarra7317 7 років тому +2

    MateMovil Suba videos de hidrostática por favor!! Tus videos son geniales!! :D

  • @jatr_swiftie2741
    @jatr_swiftie2741 5 років тому +6

    Cuando miro rubinos Se me complica fisica, pero con usted es Mas entendible, y LA Ultima parece Un problema de admision de uni jaja

  • @bayroncandanedo2785
    @bayroncandanedo2785 7 років тому +1

    estan muy buena tu esplicacion ....genial

    • @Matemovil
      @Matemovil  7 років тому

      gracias Bayron, un saludo!

  • @andresortizpuerta3098
    @andresortizpuerta3098 6 років тому +4

    Que ejercicio para largo, pero lo bueno fue que se entendió.

  • @andresmansbach8492
    @andresmansbach8492 Рік тому

    Hola Jorge cómo estás?, Jorge una consulta, pongamos un ejemplo que puede presentarse en la vida real, si van 2 vehículos en movimiento, desplazándose en el mismo sentido sobre la misma vía, supongamos que el vehículo 1 es una moto y el vehículo 2 es un automóvil, el automóvil choca a la moto, que sucede allí? Hay sumatoria de velocidades? La moto sale impulsada sobre su velocidad más la velocidad del automóvil?
    Gracias por tu aclaración

  • @marcoantoniomaccapaguzman9319
    @marcoantoniomaccapaguzman9319 7 років тому +2

    bueno tus vídeos me ayudo mucho

    • @Matemovil
      @Matemovil  7 років тому

      genial Marco, un saludo!

  • @marialopezpadilla356
    @marialopezpadilla356 4 роки тому +1

    Hola! Este mismo también podría resolverse con vectores? Con la i y la j?

  • @liss2348
    @liss2348 7 років тому +2

    ayyy un problema casi media hora pero pero muyy interesante y biem explicado
    gracias.... saludos

    • @Matemovil
      @Matemovil  7 років тому +6

      me llevo casi 2 días grabarlo. Cuando ya lo tenía listo, resulta que la cámara se colgó. Habían errores de edición. Pero finalmente, ya está arriba y eso es lo importante. Un saludo!

  • @Penovifacundo
    @Penovifacundo 7 років тому +2

    gracias me hiciste aprobar física con otros videos

    • @Matemovil
      @Matemovil  7 років тому +2

      Nop, aprobaste por TU esfuerzo. Un saludo!!!!

  • @1xjordanx
    @1xjordanx 6 років тому +1

    Profe una consulta, no saldría de manera más fácil y práctica con el método geométrico?, Ya que forman ángulo de noventa los fragmentos

  • @AnalisisBatallasdefree
    @AnalisisBatallasdefree 7 років тому +1

    Gracias gracias

  • @lelioalexanderguizapena2452
    @lelioalexanderguizapena2452 3 роки тому

    el ejercicio de los resortes su velocidad es en sentido vectorial a la izquierda y es -10¡ m/s aplicando diagrama de cuerpo libre concluimos que la velocidad de A e la mitad de B debido a que es 2 veces su masa donde la fuerza elástica aplicada es constante para ambas masas.

  • @tomas8982
    @tomas8982 5 років тому

    Movimiento relativo y transformadas de Galileo. Si usted no puedo entender nada. Es un Dios de la Física!

  • @christophergomezaguilar9401
    @christophergomezaguilar9401 7 років тому +1

    De lo mejor !

  • @luisfernadotancarapardo1662
    @luisfernadotancarapardo1662 7 років тому +2

    muy buenos videos haci da ganas de studiar matematicas jeje

    • @Matemovil
      @Matemovil  7 років тому

      genial, un saludo!!!

  • @pierorenatoespinozaaguirre8461
    @pierorenatoespinozaaguirre8461 7 років тому +8

    Dios santo eso aun no lo se XD ..... estoy mirando los antiguos no mas pero ya llegare a este video. se ve interesante c:

  • @salomevalarezo5903
    @salomevalarezo5903 7 років тому +6

    gracias por toda tu ayuda. podrías responderme porq 3 es igual a 3

    • @Matemovil
      @Matemovil  7 років тому +7

      xq no es igual a 4

    • @salomevalarezo5903
      @salomevalarezo5903 7 років тому +1

      MateMovil esa respuesta a mi profesor de física no le va a gustar😚😂

    • @Matemovil
      @Matemovil  7 років тому

      jajaja

  • @santiagozg189
    @santiagozg189 5 років тому

    brother eres un capo enseñando, eres ordenado y tú letra es de la ptm xD...por favor si puedes haz tutoriales de todo lo que son Integrales...Saludos

  • @andresdunk5994
    @andresdunk5994 7 років тому +1

    excelente el video 👌

    • @Matemovil
      @Matemovil  7 років тому +1

      gracias Andrés, un saludo!

  • @bastiancarcamo3499
    @bastiancarcamo3499 6 років тому

    Resolví el ejercicio antes de que diera la explicación de este y me dio el resultado jaja, gracias a los otros dos videos.

  • @milencalinaresmartinez5826
    @milencalinaresmartinez5826 7 років тому +2

    hola Jorge estaría buenísimo que hagas videos de cálculo vengo siguiendote y me sirven bastante tus videos

    • @Matemovil
      @Matemovil  7 років тому

      que bueno Mile, espero poder empezar pronto a subir cálculo. Un saludo!

  • @pierogomez825
    @pierogomez825 4 роки тому

    Alguien sabe si esto viene en San Marcos? Pdta;Buen video y buena explicación entendí todo

  • @mssv3560
    @mssv3560 6 років тому

    Gracias

  • @santiagopineda7808
    @santiagopineda7808 7 років тому

    profe .. puede enseñar calculo diferencial y todo lo que necesitamos saber : )

  • @felixfigueroa1794
    @felixfigueroa1794 5 місяців тому

    Dónde encuentro la solución de los ejercicios propuestos

  • @nicxi1
    @nicxi1 7 років тому +1

    Sensei Jorge! :)

  • @Azul0044
    @Azul0044 6 років тому +2

    Muy buenos tus videos! Entiendo más física viendo tus videos que yendo a la Universidad jajaja

  • @climentsll1735
    @climentsll1735 Рік тому

    que grande eres

  • @jonathanvig2453
    @jonathanvig2453 5 років тому

    Eres el mejor

    • @Matemovil
      @Matemovil  5 років тому

      Gracias Jonathan, saludos ✌️✌️

  • @danielaazucena9226
    @danielaazucena9226 4 роки тому

    excelente explicacion ,valla que larga resolucion del ultimo ejercicio , para un examen uhhhh no sabria si hacerlo :,,,v

  • @marcoantoniomaccapaguzman9319
    @marcoantoniomaccapaguzman9319 7 років тому +4

    amigo puedes factorizar 64x^3+125y^3. saludos

    • @Matemovil
      @Matemovil  7 років тому

      Hola y gracias por escribirnos. Te agradeceré colocar tu pregunta en nuestro foro, donde siempre habrá un experto dispuesto a ayudarte, nos vemos allí:
      goo.gl/7xlP1R

    • @aquilesmoreno169
      @aquilesmoreno169 6 років тому

      Eso es lo de menos amigos XD

  • @davorfiestas6592
    @davorfiestas6592 6 років тому

    Que hermosa es la física, gracias profe.

  • @santosgonzalesluisenrique617
    @santosgonzalesluisenrique617 3 роки тому

    profesor en el eje y actua la fuerza de gravedad o peso entonces por que se conserva la cantidad de movimiento en ese eje y.
    ayudeme estoy confundido .

  • @manutex2850
    @manutex2850 7 років тому

    porfaaa sube un video de vectores , pero q sean problemas con grafica(q ten den una grafica y tengas q desarrollar el.problema en la grafica) porfaa te lo ruegoo

  • @ludgerd.mendoza3035
    @ludgerd.mendoza3035 6 років тому +2

    un problema similar esta en mi ficha xd gracias

  • @ezequielmamaniaguilar
    @ezequielmamaniaguilar 3 роки тому

    Profe, acá me quedo la duda de por que se plantea la conservación de la cantidad de movimiento en el eje y.
    Osea, se sabe que si la sumatoria de fuerzas externas en alguno de los ejes es cero, entonces ahí se conserva la cantidad de movimiento.
    Pero acá, en el eje 'y' yo tengo la fuerza peso.

    • @ezequielmamaniaguilar
      @ezequielmamaniaguilar 3 роки тому

      O... se puede decir que el delta de tiempo en la cual se plantea la conservacion de la cantidad de movimiento es tan pequeño, que se desprecia esa fuerza externa?

  • @calaquita29
    @calaquita29 5 років тому

    un crack

  • @cristianrojas655
    @cristianrojas655 4 роки тому

    Desarrollando sólo en el eje x se obtiene el resultado de v1 + v2, se divide en dos y se llega a la respuesta más rápido

  • @hablalealmundo7450
    @hablalealmundo7450 4 роки тому

    Disculpa el ejercicio depende mucho de calcular el seno y el coseno como fraccionarios, pero hacerlo no es tan sencillo, me da otros fraccionarios diferentes. Tienes alguna tabla que podamos usar para esas transformaciones. Seria de mucha ayuda. Gracias

    • @lonelystar1214
      @lonelystar1214 3 роки тому

      Siempre es más fácil con fracciones. Jorge si subió una tabla de ángulos notables yo lo descargué si no mal recuerdo está en la parte de vectores.
      Aunq de todas formas hay cientos de tablas en internet

  • @karendianaalarconcriollo2383
    @karendianaalarconcriollo2383 6 років тому

    Disculpe, pero en el ejercicio "5", mi respuesta no coincide con la suya, no se si podrá responder este sms.
    Los datos que utilice son: masa del cañón, 1000kg; masa del proyectil, 0.5 kg; velocidad del proyectil 100 m/s. Con lo cual la velocidad me sale 20 m/s.
    pero a usted le salió 0.05m/s. Por favor podría despejar mi duda...

    • @Azul0044
      @Azul0044 6 років тому +1

      karen diana alarcon criollo tenés que hacer Po=Pf, Velocidad inicial del proyectil y camión es Cero porque estan en reposo. Velocidad final del proyectil 100m/s
      mcañon.Voc+mproyectil+Vop= mc.Vfc+mp.Vfp. Te queda reemplazar a mí me da Vfinaldelcañon= -0,05m/s pero como pide rapidez solo te queda 0,05m/s

  • @marcelogn6225
    @marcelogn6225 7 років тому +1

    Y todavía dejas tarea ! te mamaste jajajja que buen profe csm sigue asi crack

  • @bryancardenas1193
    @bryancardenas1193 7 років тому +1

    Ya me salio las respuestas

  • @NamelessGamer69
    @NamelessGamer69 5 років тому

    v2Y no es negativa?

  • @marcelocostantino306
    @marcelocostantino306 3 роки тому

    Venía bien hasta que metió la sarasa de la constante universal. Con remplazar la expresión de v1 en términos de v2 que había obtenido antes alcanzaba.

  • @Azul0044
    @Azul0044 6 років тому +3

    Alguien pudo hacer el problema 7? No me sale

    • @javierbayter1870
      @javierbayter1870 5 років тому +1

      p1=p2 => 2m.vA=m.vB = 2vA=20m/s => vA=10m/s

  • @marco_m_
    @marco_m_ 6 років тому

    hola, tengo una duda 12:40 porque es v1 / v2 y no al reves

    • @lonelystar1214
      @lonelystar1214 3 роки тому

      Te va a dar los mismos valores si lo haces al revés

  • @lelioalexanderguizapena2452
    @lelioalexanderguizapena2452 3 роки тому

    El primero la velocidad es negativa y es -0.05m/s

  • @eduardoroldan5846
    @eduardoroldan5846 7 років тому +2

    parecido al de la uni

    • @Matemovil
      @Matemovil  7 років тому

      sip, este problema estaba muy interesante!

  • @andrestorrez8845
    @andrestorrez8845 6 років тому

    Profee... v3 = me sale 65.08m/s 😕

  • @luisfernadotancarapardo1662
    @luisfernadotancarapardo1662 7 років тому +1

    2do comentario siiuuu

  • @Rosy0829
    @Rosy0829 4 роки тому

    ¿Por qué cos 37° da ⅘

    • @lonelystar1214
      @lonelystar1214 3 роки тому

      Es un angulo notable. La calculadora te dará 0.79863551 lo q se hace es redondear a 0.8 que es igual a 4/5.
      Existen tablas de ángulos notables. Y el profe Jorge también los subió...están en la parte de vectores

  • @valerianahuelpan
    @valerianahuelpan 7 років тому

    Es el ultimo nivel? :((

  • @elogro7854
    @elogro7854 3 роки тому

    me toman esto en mi parcial y me muero

  • @juanhanccobaldeon7585
    @juanhanccobaldeon7585 3 роки тому +1

    todo bien, pero explicas muy lento :c

    • @N1Noob
      @N1Noob 3 роки тому +1

      Puedes cambiarle la velocidad y verlo a x2 es así como yo lo hago

    • @lonelystar1214
      @lonelystar1214 3 роки тому +1

      Lento y seguro es mejor

  • @victorrodriguez5304
    @victorrodriguez5304 7 років тому +1

    4 comentario creo! :v

  • @itsmehe5344
    @itsmehe5344 5 років тому

    problema tipo NASA? v:

  • @paulhernandez112
    @paulhernandez112 4 роки тому

    Eres dios acaso?